Những câu hỏi liên quan
dia fic
Xem chi tiết
Akai Haruma
4 tháng 1 2021 lúc 19:08

Lời giải:

Tìm min:

Áp dụng BĐT AM-GM:

$x^2+y^2+z^2\geq \frac{(x+y+z)^2}{3}=\frac{6^2}{3}=12$

Vậy $A_{\min}=12$. Giá trị này đạt tại $x=y=z=2$

--------------

Tìm max:

$A=x^2+y^2+z^2=(x+y+z)^2-2(xy+yz+xz)=36-2(xy+yz+xz)$

Vì $x,y,z\geq 0\Rightarrow xy+yz+xz\geq 0$

$\Rightarrow A=36-2(xy+yz+xz)\leq 36$

Vậy $A_{\max}=36$. Giá trị này đạt tại $(x,y,z)=(0,0,6)$ và hoán vị.

Bình luận (0)
Big City Boy
Xem chi tiết
Big City Boy
Xem chi tiết
Hồ Thị Mai Linh
Xem chi tiết
Kiệt Nguyễn
Xem chi tiết
zZz Cool Kid_new zZz
6 tháng 8 2020 lúc 9:48

Cay, đánh xong rồi tự nhiên bấm hủy :v

Ta có:\(x+y+z=xyz\Leftrightarrow\frac{1}{xy}+\frac{1}{yz}+\frac{1}{zx}=1\)

Đặt \(\left(\frac{1}{x};\frac{1}{y};\frac{1}{z}\right)\rightarrow\left(a;b;c\right)\Rightarrow ab+bc+ca=1\)

Khi đó:

\(A=\frac{a^2\left(1+2b\right)}{b}+\frac{b^2\left(1+2c\right)}{c}+\frac{c^2\left(1+2a\right)}{a}\)

\(=\frac{a^2}{b}+\frac{b^2}{c}+\frac{c^2}{a}+2\left(a^2+b^2+c^2\right)\)

\(\ge\frac{\left(a+b+c\right)^2}{a+b+c}+2\cdot\frac{\left(a+b+c\right)^2}{3}\)

\(=a+b+c+\frac{2\left(a+b+c\right)^2}{3}\)

\(\ge\sqrt{3\left(ab+bc+ca\right)}+\frac{6\left(ab+bc+ca\right)}{3}\)

\(=2+\sqrt{3}\)

Đẳng thức xảy ra tại \(x=y=z=\sqrt{3}\)

Bình luận (0)
 Khách vãng lai đã xóa
Trí Tiên亗
6 tháng 8 2020 lúc 9:58

zZz Cool Kid_new zZz. Sai đề rồi bạn êii !

Nếu bạn đặt như vậy thì 

\(A=\frac{y-2}{x^2}+\frac{z-2}{y^2}+\frac{x-2}{z^2}\)

\(=\frac{a^2\left(1-2b\right)}{b}+\frac{b^2\left(1-2c\right)}{c}+\frac{c^2\left(1-2a\right)}{a}\)

\(=\frac{a^2}{b}+\frac{b^2}{c}+\frac{c^2}{a}-2.\left(a^2+b^2+c^2\right)\)

Bình luận (0)
 Khách vãng lai đã xóa
zZz Cool Kid_new zZz
6 tháng 8 2020 lúc 10:13

thấy nó sương sương đề thanh hóa năm nay nên t dựa theo đề kia làm luôn :3 

Bình luận (0)
 Khách vãng lai đã xóa
Angela jolie
Xem chi tiết
Nguyễn Việt Lâm
31 tháng 5 2020 lúc 13:19

Áp dụng BĐT:

\(xyz\ge\left(x+y-z\right)\left(y+z-x\right)\left(x+z-y\right)\)

\(\Leftrightarrow xyz\ge\left(1-2x\right)\left(1-2y\right)\left(1-2z\right)\)

\(\Leftrightarrow xyz\ge1+4\left(xy+yz+zx\right)-2\left(x+y+z\right)-8xyz\)

\(\Leftrightarrow9xyz\ge4\left(xy+yz+zx\right)-1\)

\(\Rightarrow P=x^2+y^2+z^2+\frac{9}{2}xyz\ge x^2+y^2+z^2+2\left(xy+yz+zx\right)-\frac{1}{2}\)

\(\Leftrightarrow P\ge\left(x+y+z\right)^2-\frac{1}{2}=\frac{1}{2}\)

Dấu "=" xảy ra khi \(x=y=z=\frac{1}{3}\)

Lại có:

\(xy+yz+zx=\left(xy+yz+zx\right)\left(x+y+z\right)\ge3\sqrt[3]{x^2y^2z^2}.3\sqrt[3]{xyz}=9xyz\)

\(\Rightarrow P\le x^2+y^2+z^2+\frac{1}{2}\left(xy+yz+zx\right)\)

\(P\le\left(x+y+z\right)^2-\frac{3}{2}\left(xy+yz+zx\right)\le\left(x+y+z\right)^2=1\)

Dấu "=" xảy ra khi \(\left(x;y;z\right)=\left(0;0;1\right)\) và hoán vị

Bình luận (0)
DTD2006ok
Xem chi tiết
Nguyễn Việt Lâm
5 tháng 1 2021 lúc 18:27

Bài này chỉ có min, không có max của A nhé bạn

Muốn có max thì x;y;z phải không âm

Bình luận (0)
Nguyen Thi Bich Huong
Xem chi tiết
Nguyễn Việt Lâm
21 tháng 1 2021 lúc 10:59

\(2=x^2+y^2+z^2\ge y^2+z^2\ge2yz\Rightarrow yz\le1\)

\(P=x\left(1-yz\right)+y+z\Rightarrow P^2\le\left[x^2+\left(y+z\right)^2\right]\left[\left(1-yz\right)^2+1\right]\)

\(P^2\le\left(2+2yz\right)\left(y^2z^2-2yz+2\right)\)

\(P^2\le2\left(yz\right)^3-2\left(yz\right)^2+4=2y^2z^2\left(yz-1\right)+4\le4\)

\(\Rightarrow P\le2\)

\(P_{max}=2\) khi \(\left(x;y;z\right)=\left(0;1;1\right)\) và các hoán vị

Bình luận (0)
ABC
Xem chi tiết